Froebenius norm is unitarily invariant.












6












$begingroup$


I'm considering the norm defined on matrices by



$$|A|_F = sqrt{sum_{i,j}|a_{ij}|^2}$$



I want to show that it is unitarily invariant, so that for unitary $U$ we have that



$$|UA|_F = |A|_F = |AU|_F$$



however I have trouble doing it directly. Writing $|UA|_F$ directly I find by Cauchy-Schwarz that



$$|UA|_F = sqrt{sum_{i,j}left|sum_{k=1}^{n}u_{ik}a_{kj}right|^2}= sqrt{sum_{i,j}|langle U_i,overline{A_j}rangle|^2}leq sqrt{sum_{i,j}|A_j|^2}$$



where $U_i$ denotes the $i$th row of $U$ and $A_j$ the $j$th column of $A$. However this estimate is to crude and will not equal $|A|_F$. I would like to prove this without refering to trace or singular values and would appreciate a hint, rather than a full solution, on how to tackle this problem.





EDIT: Completion of the proof based on the answer from $A.Gamma$:



Since the rows of $U$ constitute an orthonormal basis for $mathbb{C}^n$ we find by Parsevals theorem that



$$|UA_j|_2^2 = sum_{i=1}^{n}left|sum_{k=1}^{n}u_{ik}a_{kj}right|^2 = sum_{i=1}^{n}|langle U_i,overline{A_j}rangle|^2 = |overline{A_j}|_2^2 = |A_j|_2^2$$










share|cite|improve this question











$endgroup$

















    6












    $begingroup$


    I'm considering the norm defined on matrices by



    $$|A|_F = sqrt{sum_{i,j}|a_{ij}|^2}$$



    I want to show that it is unitarily invariant, so that for unitary $U$ we have that



    $$|UA|_F = |A|_F = |AU|_F$$



    however I have trouble doing it directly. Writing $|UA|_F$ directly I find by Cauchy-Schwarz that



    $$|UA|_F = sqrt{sum_{i,j}left|sum_{k=1}^{n}u_{ik}a_{kj}right|^2}= sqrt{sum_{i,j}|langle U_i,overline{A_j}rangle|^2}leq sqrt{sum_{i,j}|A_j|^2}$$



    where $U_i$ denotes the $i$th row of $U$ and $A_j$ the $j$th column of $A$. However this estimate is to crude and will not equal $|A|_F$. I would like to prove this without refering to trace or singular values and would appreciate a hint, rather than a full solution, on how to tackle this problem.





    EDIT: Completion of the proof based on the answer from $A.Gamma$:



    Since the rows of $U$ constitute an orthonormal basis for $mathbb{C}^n$ we find by Parsevals theorem that



    $$|UA_j|_2^2 = sum_{i=1}^{n}left|sum_{k=1}^{n}u_{ik}a_{kj}right|^2 = sum_{i=1}^{n}|langle U_i,overline{A_j}rangle|^2 = |overline{A_j}|_2^2 = |A_j|_2^2$$










    share|cite|improve this question











    $endgroup$















      6












      6








      6


      1



      $begingroup$


      I'm considering the norm defined on matrices by



      $$|A|_F = sqrt{sum_{i,j}|a_{ij}|^2}$$



      I want to show that it is unitarily invariant, so that for unitary $U$ we have that



      $$|UA|_F = |A|_F = |AU|_F$$



      however I have trouble doing it directly. Writing $|UA|_F$ directly I find by Cauchy-Schwarz that



      $$|UA|_F = sqrt{sum_{i,j}left|sum_{k=1}^{n}u_{ik}a_{kj}right|^2}= sqrt{sum_{i,j}|langle U_i,overline{A_j}rangle|^2}leq sqrt{sum_{i,j}|A_j|^2}$$



      where $U_i$ denotes the $i$th row of $U$ and $A_j$ the $j$th column of $A$. However this estimate is to crude and will not equal $|A|_F$. I would like to prove this without refering to trace or singular values and would appreciate a hint, rather than a full solution, on how to tackle this problem.





      EDIT: Completion of the proof based on the answer from $A.Gamma$:



      Since the rows of $U$ constitute an orthonormal basis for $mathbb{C}^n$ we find by Parsevals theorem that



      $$|UA_j|_2^2 = sum_{i=1}^{n}left|sum_{k=1}^{n}u_{ik}a_{kj}right|^2 = sum_{i=1}^{n}|langle U_i,overline{A_j}rangle|^2 = |overline{A_j}|_2^2 = |A_j|_2^2$$










      share|cite|improve this question











      $endgroup$




      I'm considering the norm defined on matrices by



      $$|A|_F = sqrt{sum_{i,j}|a_{ij}|^2}$$



      I want to show that it is unitarily invariant, so that for unitary $U$ we have that



      $$|UA|_F = |A|_F = |AU|_F$$



      however I have trouble doing it directly. Writing $|UA|_F$ directly I find by Cauchy-Schwarz that



      $$|UA|_F = sqrt{sum_{i,j}left|sum_{k=1}^{n}u_{ik}a_{kj}right|^2}= sqrt{sum_{i,j}|langle U_i,overline{A_j}rangle|^2}leq sqrt{sum_{i,j}|A_j|^2}$$



      where $U_i$ denotes the $i$th row of $U$ and $A_j$ the $j$th column of $A$. However this estimate is to crude and will not equal $|A|_F$. I would like to prove this without refering to trace or singular values and would appreciate a hint, rather than a full solution, on how to tackle this problem.





      EDIT: Completion of the proof based on the answer from $A.Gamma$:



      Since the rows of $U$ constitute an orthonormal basis for $mathbb{C}^n$ we find by Parsevals theorem that



      $$|UA_j|_2^2 = sum_{i=1}^{n}left|sum_{k=1}^{n}u_{ik}a_{kj}right|^2 = sum_{i=1}^{n}|langle U_i,overline{A_j}rangle|^2 = |overline{A_j}|_2^2 = |A_j|_2^2$$







      linear-algebra matrices norm






      share|cite|improve this question















      share|cite|improve this question













      share|cite|improve this question




      share|cite|improve this question








      edited Jan 4 at 12:39







      Olof Rubin

















      asked Jan 4 at 12:03









      Olof RubinOlof Rubin

      1,131316




      1,131316






















          2 Answers
          2






          active

          oldest

          votes


















          5












          $begingroup$

          Since
          $$
          UA=[UA_1 UA_2 ldots UA_n]
          $$

          you need to prove that
          $$
          |UA|_F^2=sum_{j=1}^n|UA_j|_2^2stackrel{?}{=}sum_{j=1}^n|A_j|_2^2=|A|_F^2.
          $$

          It suffice to prove that $|UA_j|_2^2=|A_j|_2^2$.



          P.S. For $AU$ use conjugation.






          share|cite|improve this answer









          $endgroup$













          • $begingroup$
            I see we can use that the rows of $U$ constitute an ONB for $mathbb{C}^n$, I completed my proof above using your recommendation. Thank you. I realise now that part of my confusion came from the fact that unitary matrices don't need to have norm $1$.
            $endgroup$
            – Olof Rubin
            Jan 4 at 12:40








          • 2




            $begingroup$
            @OlofRubin Alternatively, one can use that a unitary matrix is isometric, i.e. $|Ux|_2=|x|_2$. Proof: $$|Ux|_2^2=(Ux)^*Ux=x^*underbrace{U^*U}_{=I}x=x^*x=|x|_2^2.$$
            $endgroup$
            – A.Γ.
            Jan 4 at 12:46










          • $begingroup$
            Ah yes that is more direct.
            $endgroup$
            – Olof Rubin
            Jan 4 at 13:26



















          4












          $begingroup$

          Quick and dirty:



          $$|UA|_F^2 = operatorname{Tr}((UA)^*(UA)) = operatorname{Tr}(A^*U^*UA) = operatorname{Tr}(A^*A) = |A|_F^2$$
          and then since $U^*$ is also unitary
          $$|AU|_F^2 = |(U^*A^*)^*|_F^2 = |U^*A^*|_F^2 = |A^*|_F^2 = |A|_F^2$$





          Alternative argument:



          Note that $A^*A ge 0$ so there exists an orthonormal basis ${u_1, ldots, u_n}$ for $mathbb{C}^n$ such that $A^*A u_i = lambda_i u_i$ for some $lambda ge 0$.



          We have
          $$sum_{i=1}^n |Au_i|_2^2 = sum_{i=1}^n langle Au_i, Au_irangle = sum_{i=1}^n langle A^*Au_i, u_irangle = sum_{i=1}^n lambda_i =operatorname{Tr}(A^*A) = |A|_F^2$$



          because the trace is the sum of eigenvalues.



          The interesting part is that the sum $sum_{i=1}^n |Au_i|_2^2$ is actually independent of the choice of the orthonormal basis ${u_1, ldots, u_n}$. Indeed, if ${v_1, ldots, v_n}$ is some other orthonormal basis for $mathbb{C}^n$, we have
          begin{align}
          sum_{i=1}^n |Au_i|_2^2 &= sum_{i=1}^n langle A^*Au_i, u_irangle\
          &= sum_{i=1}^n leftlangle sum_{j=1}^nlangle u_i,v_jrangle A^*A v_j , sum_{k=1}^nlangle u_i,v_krangle v_krightrangle\
          &= sum_{j=1}^n sum_{k=1}^n left(sum_{i=1}^nlangle u_i,v_jrangle langle v_k,u_irangleright)langle A^*A v_j,v_krangle\
          &= sum_{j=1}^n sum_{k=1}^n langle v_j,v_kranglelangle A^*A v_j,v_krangle\
          &= sum_{j=1}^n langle A^*A v_j,v_jrangle\
          &= sum_{j=1}^n |Av_j|_2^2
          end{align}



          Now, if $U$ is unitary, for any orthonormal basis ${u_1, ldots, u_n}$ we have that ${Uu_1, ldots, Uu_n}$ is also an orthonormal basis so:



          $$|AU|_F^2 = sum_{i=1}^n |A(Ue_i)|^2 = |A|_F^2$$






          share|cite|improve this answer











          $endgroup$













          • $begingroup$
            That's true, but the OP doesn't want to refer to trace.
            $endgroup$
            – A.Γ.
            Jan 4 at 12:26












          • $begingroup$
            @A.Γ. True, I missed that. Your argument is great. I have added an alternative argument which may be helpful, but it still uses the trace.
            $endgroup$
            – mechanodroid
            Jan 4 at 12:30











          Your Answer





          StackExchange.ifUsing("editor", function () {
          return StackExchange.using("mathjaxEditing", function () {
          StackExchange.MarkdownEditor.creationCallbacks.add(function (editor, postfix) {
          StackExchange.mathjaxEditing.prepareWmdForMathJax(editor, postfix, [["$", "$"], ["\\(","\\)"]]);
          });
          });
          }, "mathjax-editing");

          StackExchange.ready(function() {
          var channelOptions = {
          tags: "".split(" "),
          id: "69"
          };
          initTagRenderer("".split(" "), "".split(" "), channelOptions);

          StackExchange.using("externalEditor", function() {
          // Have to fire editor after snippets, if snippets enabled
          if (StackExchange.settings.snippets.snippetsEnabled) {
          StackExchange.using("snippets", function() {
          createEditor();
          });
          }
          else {
          createEditor();
          }
          });

          function createEditor() {
          StackExchange.prepareEditor({
          heartbeatType: 'answer',
          autoActivateHeartbeat: false,
          convertImagesToLinks: true,
          noModals: true,
          showLowRepImageUploadWarning: true,
          reputationToPostImages: 10,
          bindNavPrevention: true,
          postfix: "",
          imageUploader: {
          brandingHtml: "Powered by u003ca class="icon-imgur-white" href="https://imgur.com/"u003eu003c/au003e",
          contentPolicyHtml: "User contributions licensed under u003ca href="https://creativecommons.org/licenses/by-sa/3.0/"u003ecc by-sa 3.0 with attribution requiredu003c/au003e u003ca href="https://stackoverflow.com/legal/content-policy"u003e(content policy)u003c/au003e",
          allowUrls: true
          },
          noCode: true, onDemand: true,
          discardSelector: ".discard-answer"
          ,immediatelyShowMarkdownHelp:true
          });


          }
          });














          draft saved

          draft discarded


















          StackExchange.ready(
          function () {
          StackExchange.openid.initPostLogin('.new-post-login', 'https%3a%2f%2fmath.stackexchange.com%2fquestions%2f3061577%2ffroebenius-norm-is-unitarily-invariant%23new-answer', 'question_page');
          }
          );

          Post as a guest















          Required, but never shown

























          2 Answers
          2






          active

          oldest

          votes








          2 Answers
          2






          active

          oldest

          votes









          active

          oldest

          votes






          active

          oldest

          votes









          5












          $begingroup$

          Since
          $$
          UA=[UA_1 UA_2 ldots UA_n]
          $$

          you need to prove that
          $$
          |UA|_F^2=sum_{j=1}^n|UA_j|_2^2stackrel{?}{=}sum_{j=1}^n|A_j|_2^2=|A|_F^2.
          $$

          It suffice to prove that $|UA_j|_2^2=|A_j|_2^2$.



          P.S. For $AU$ use conjugation.






          share|cite|improve this answer









          $endgroup$













          • $begingroup$
            I see we can use that the rows of $U$ constitute an ONB for $mathbb{C}^n$, I completed my proof above using your recommendation. Thank you. I realise now that part of my confusion came from the fact that unitary matrices don't need to have norm $1$.
            $endgroup$
            – Olof Rubin
            Jan 4 at 12:40








          • 2




            $begingroup$
            @OlofRubin Alternatively, one can use that a unitary matrix is isometric, i.e. $|Ux|_2=|x|_2$. Proof: $$|Ux|_2^2=(Ux)^*Ux=x^*underbrace{U^*U}_{=I}x=x^*x=|x|_2^2.$$
            $endgroup$
            – A.Γ.
            Jan 4 at 12:46










          • $begingroup$
            Ah yes that is more direct.
            $endgroup$
            – Olof Rubin
            Jan 4 at 13:26
















          5












          $begingroup$

          Since
          $$
          UA=[UA_1 UA_2 ldots UA_n]
          $$

          you need to prove that
          $$
          |UA|_F^2=sum_{j=1}^n|UA_j|_2^2stackrel{?}{=}sum_{j=1}^n|A_j|_2^2=|A|_F^2.
          $$

          It suffice to prove that $|UA_j|_2^2=|A_j|_2^2$.



          P.S. For $AU$ use conjugation.






          share|cite|improve this answer









          $endgroup$













          • $begingroup$
            I see we can use that the rows of $U$ constitute an ONB for $mathbb{C}^n$, I completed my proof above using your recommendation. Thank you. I realise now that part of my confusion came from the fact that unitary matrices don't need to have norm $1$.
            $endgroup$
            – Olof Rubin
            Jan 4 at 12:40








          • 2




            $begingroup$
            @OlofRubin Alternatively, one can use that a unitary matrix is isometric, i.e. $|Ux|_2=|x|_2$. Proof: $$|Ux|_2^2=(Ux)^*Ux=x^*underbrace{U^*U}_{=I}x=x^*x=|x|_2^2.$$
            $endgroup$
            – A.Γ.
            Jan 4 at 12:46










          • $begingroup$
            Ah yes that is more direct.
            $endgroup$
            – Olof Rubin
            Jan 4 at 13:26














          5












          5








          5





          $begingroup$

          Since
          $$
          UA=[UA_1 UA_2 ldots UA_n]
          $$

          you need to prove that
          $$
          |UA|_F^2=sum_{j=1}^n|UA_j|_2^2stackrel{?}{=}sum_{j=1}^n|A_j|_2^2=|A|_F^2.
          $$

          It suffice to prove that $|UA_j|_2^2=|A_j|_2^2$.



          P.S. For $AU$ use conjugation.






          share|cite|improve this answer









          $endgroup$



          Since
          $$
          UA=[UA_1 UA_2 ldots UA_n]
          $$

          you need to prove that
          $$
          |UA|_F^2=sum_{j=1}^n|UA_j|_2^2stackrel{?}{=}sum_{j=1}^n|A_j|_2^2=|A|_F^2.
          $$

          It suffice to prove that $|UA_j|_2^2=|A_j|_2^2$.



          P.S. For $AU$ use conjugation.







          share|cite|improve this answer












          share|cite|improve this answer



          share|cite|improve this answer










          answered Jan 4 at 12:13









          A.Γ.A.Γ.

          22.7k32656




          22.7k32656












          • $begingroup$
            I see we can use that the rows of $U$ constitute an ONB for $mathbb{C}^n$, I completed my proof above using your recommendation. Thank you. I realise now that part of my confusion came from the fact that unitary matrices don't need to have norm $1$.
            $endgroup$
            – Olof Rubin
            Jan 4 at 12:40








          • 2




            $begingroup$
            @OlofRubin Alternatively, one can use that a unitary matrix is isometric, i.e. $|Ux|_2=|x|_2$. Proof: $$|Ux|_2^2=(Ux)^*Ux=x^*underbrace{U^*U}_{=I}x=x^*x=|x|_2^2.$$
            $endgroup$
            – A.Γ.
            Jan 4 at 12:46










          • $begingroup$
            Ah yes that is more direct.
            $endgroup$
            – Olof Rubin
            Jan 4 at 13:26


















          • $begingroup$
            I see we can use that the rows of $U$ constitute an ONB for $mathbb{C}^n$, I completed my proof above using your recommendation. Thank you. I realise now that part of my confusion came from the fact that unitary matrices don't need to have norm $1$.
            $endgroup$
            – Olof Rubin
            Jan 4 at 12:40








          • 2




            $begingroup$
            @OlofRubin Alternatively, one can use that a unitary matrix is isometric, i.e. $|Ux|_2=|x|_2$. Proof: $$|Ux|_2^2=(Ux)^*Ux=x^*underbrace{U^*U}_{=I}x=x^*x=|x|_2^2.$$
            $endgroup$
            – A.Γ.
            Jan 4 at 12:46










          • $begingroup$
            Ah yes that is more direct.
            $endgroup$
            – Olof Rubin
            Jan 4 at 13:26
















          $begingroup$
          I see we can use that the rows of $U$ constitute an ONB for $mathbb{C}^n$, I completed my proof above using your recommendation. Thank you. I realise now that part of my confusion came from the fact that unitary matrices don't need to have norm $1$.
          $endgroup$
          – Olof Rubin
          Jan 4 at 12:40






          $begingroup$
          I see we can use that the rows of $U$ constitute an ONB for $mathbb{C}^n$, I completed my proof above using your recommendation. Thank you. I realise now that part of my confusion came from the fact that unitary matrices don't need to have norm $1$.
          $endgroup$
          – Olof Rubin
          Jan 4 at 12:40






          2




          2




          $begingroup$
          @OlofRubin Alternatively, one can use that a unitary matrix is isometric, i.e. $|Ux|_2=|x|_2$. Proof: $$|Ux|_2^2=(Ux)^*Ux=x^*underbrace{U^*U}_{=I}x=x^*x=|x|_2^2.$$
          $endgroup$
          – A.Γ.
          Jan 4 at 12:46




          $begingroup$
          @OlofRubin Alternatively, one can use that a unitary matrix is isometric, i.e. $|Ux|_2=|x|_2$. Proof: $$|Ux|_2^2=(Ux)^*Ux=x^*underbrace{U^*U}_{=I}x=x^*x=|x|_2^2.$$
          $endgroup$
          – A.Γ.
          Jan 4 at 12:46












          $begingroup$
          Ah yes that is more direct.
          $endgroup$
          – Olof Rubin
          Jan 4 at 13:26




          $begingroup$
          Ah yes that is more direct.
          $endgroup$
          – Olof Rubin
          Jan 4 at 13:26











          4












          $begingroup$

          Quick and dirty:



          $$|UA|_F^2 = operatorname{Tr}((UA)^*(UA)) = operatorname{Tr}(A^*U^*UA) = operatorname{Tr}(A^*A) = |A|_F^2$$
          and then since $U^*$ is also unitary
          $$|AU|_F^2 = |(U^*A^*)^*|_F^2 = |U^*A^*|_F^2 = |A^*|_F^2 = |A|_F^2$$





          Alternative argument:



          Note that $A^*A ge 0$ so there exists an orthonormal basis ${u_1, ldots, u_n}$ for $mathbb{C}^n$ such that $A^*A u_i = lambda_i u_i$ for some $lambda ge 0$.



          We have
          $$sum_{i=1}^n |Au_i|_2^2 = sum_{i=1}^n langle Au_i, Au_irangle = sum_{i=1}^n langle A^*Au_i, u_irangle = sum_{i=1}^n lambda_i =operatorname{Tr}(A^*A) = |A|_F^2$$



          because the trace is the sum of eigenvalues.



          The interesting part is that the sum $sum_{i=1}^n |Au_i|_2^2$ is actually independent of the choice of the orthonormal basis ${u_1, ldots, u_n}$. Indeed, if ${v_1, ldots, v_n}$ is some other orthonormal basis for $mathbb{C}^n$, we have
          begin{align}
          sum_{i=1}^n |Au_i|_2^2 &= sum_{i=1}^n langle A^*Au_i, u_irangle\
          &= sum_{i=1}^n leftlangle sum_{j=1}^nlangle u_i,v_jrangle A^*A v_j , sum_{k=1}^nlangle u_i,v_krangle v_krightrangle\
          &= sum_{j=1}^n sum_{k=1}^n left(sum_{i=1}^nlangle u_i,v_jrangle langle v_k,u_irangleright)langle A^*A v_j,v_krangle\
          &= sum_{j=1}^n sum_{k=1}^n langle v_j,v_kranglelangle A^*A v_j,v_krangle\
          &= sum_{j=1}^n langle A^*A v_j,v_jrangle\
          &= sum_{j=1}^n |Av_j|_2^2
          end{align}



          Now, if $U$ is unitary, for any orthonormal basis ${u_1, ldots, u_n}$ we have that ${Uu_1, ldots, Uu_n}$ is also an orthonormal basis so:



          $$|AU|_F^2 = sum_{i=1}^n |A(Ue_i)|^2 = |A|_F^2$$






          share|cite|improve this answer











          $endgroup$













          • $begingroup$
            That's true, but the OP doesn't want to refer to trace.
            $endgroup$
            – A.Γ.
            Jan 4 at 12:26












          • $begingroup$
            @A.Γ. True, I missed that. Your argument is great. I have added an alternative argument which may be helpful, but it still uses the trace.
            $endgroup$
            – mechanodroid
            Jan 4 at 12:30
















          4












          $begingroup$

          Quick and dirty:



          $$|UA|_F^2 = operatorname{Tr}((UA)^*(UA)) = operatorname{Tr}(A^*U^*UA) = operatorname{Tr}(A^*A) = |A|_F^2$$
          and then since $U^*$ is also unitary
          $$|AU|_F^2 = |(U^*A^*)^*|_F^2 = |U^*A^*|_F^2 = |A^*|_F^2 = |A|_F^2$$





          Alternative argument:



          Note that $A^*A ge 0$ so there exists an orthonormal basis ${u_1, ldots, u_n}$ for $mathbb{C}^n$ such that $A^*A u_i = lambda_i u_i$ for some $lambda ge 0$.



          We have
          $$sum_{i=1}^n |Au_i|_2^2 = sum_{i=1}^n langle Au_i, Au_irangle = sum_{i=1}^n langle A^*Au_i, u_irangle = sum_{i=1}^n lambda_i =operatorname{Tr}(A^*A) = |A|_F^2$$



          because the trace is the sum of eigenvalues.



          The interesting part is that the sum $sum_{i=1}^n |Au_i|_2^2$ is actually independent of the choice of the orthonormal basis ${u_1, ldots, u_n}$. Indeed, if ${v_1, ldots, v_n}$ is some other orthonormal basis for $mathbb{C}^n$, we have
          begin{align}
          sum_{i=1}^n |Au_i|_2^2 &= sum_{i=1}^n langle A^*Au_i, u_irangle\
          &= sum_{i=1}^n leftlangle sum_{j=1}^nlangle u_i,v_jrangle A^*A v_j , sum_{k=1}^nlangle u_i,v_krangle v_krightrangle\
          &= sum_{j=1}^n sum_{k=1}^n left(sum_{i=1}^nlangle u_i,v_jrangle langle v_k,u_irangleright)langle A^*A v_j,v_krangle\
          &= sum_{j=1}^n sum_{k=1}^n langle v_j,v_kranglelangle A^*A v_j,v_krangle\
          &= sum_{j=1}^n langle A^*A v_j,v_jrangle\
          &= sum_{j=1}^n |Av_j|_2^2
          end{align}



          Now, if $U$ is unitary, for any orthonormal basis ${u_1, ldots, u_n}$ we have that ${Uu_1, ldots, Uu_n}$ is also an orthonormal basis so:



          $$|AU|_F^2 = sum_{i=1}^n |A(Ue_i)|^2 = |A|_F^2$$






          share|cite|improve this answer











          $endgroup$













          • $begingroup$
            That's true, but the OP doesn't want to refer to trace.
            $endgroup$
            – A.Γ.
            Jan 4 at 12:26












          • $begingroup$
            @A.Γ. True, I missed that. Your argument is great. I have added an alternative argument which may be helpful, but it still uses the trace.
            $endgroup$
            – mechanodroid
            Jan 4 at 12:30














          4












          4








          4





          $begingroup$

          Quick and dirty:



          $$|UA|_F^2 = operatorname{Tr}((UA)^*(UA)) = operatorname{Tr}(A^*U^*UA) = operatorname{Tr}(A^*A) = |A|_F^2$$
          and then since $U^*$ is also unitary
          $$|AU|_F^2 = |(U^*A^*)^*|_F^2 = |U^*A^*|_F^2 = |A^*|_F^2 = |A|_F^2$$





          Alternative argument:



          Note that $A^*A ge 0$ so there exists an orthonormal basis ${u_1, ldots, u_n}$ for $mathbb{C}^n$ such that $A^*A u_i = lambda_i u_i$ for some $lambda ge 0$.



          We have
          $$sum_{i=1}^n |Au_i|_2^2 = sum_{i=1}^n langle Au_i, Au_irangle = sum_{i=1}^n langle A^*Au_i, u_irangle = sum_{i=1}^n lambda_i =operatorname{Tr}(A^*A) = |A|_F^2$$



          because the trace is the sum of eigenvalues.



          The interesting part is that the sum $sum_{i=1}^n |Au_i|_2^2$ is actually independent of the choice of the orthonormal basis ${u_1, ldots, u_n}$. Indeed, if ${v_1, ldots, v_n}$ is some other orthonormal basis for $mathbb{C}^n$, we have
          begin{align}
          sum_{i=1}^n |Au_i|_2^2 &= sum_{i=1}^n langle A^*Au_i, u_irangle\
          &= sum_{i=1}^n leftlangle sum_{j=1}^nlangle u_i,v_jrangle A^*A v_j , sum_{k=1}^nlangle u_i,v_krangle v_krightrangle\
          &= sum_{j=1}^n sum_{k=1}^n left(sum_{i=1}^nlangle u_i,v_jrangle langle v_k,u_irangleright)langle A^*A v_j,v_krangle\
          &= sum_{j=1}^n sum_{k=1}^n langle v_j,v_kranglelangle A^*A v_j,v_krangle\
          &= sum_{j=1}^n langle A^*A v_j,v_jrangle\
          &= sum_{j=1}^n |Av_j|_2^2
          end{align}



          Now, if $U$ is unitary, for any orthonormal basis ${u_1, ldots, u_n}$ we have that ${Uu_1, ldots, Uu_n}$ is also an orthonormal basis so:



          $$|AU|_F^2 = sum_{i=1}^n |A(Ue_i)|^2 = |A|_F^2$$






          share|cite|improve this answer











          $endgroup$



          Quick and dirty:



          $$|UA|_F^2 = operatorname{Tr}((UA)^*(UA)) = operatorname{Tr}(A^*U^*UA) = operatorname{Tr}(A^*A) = |A|_F^2$$
          and then since $U^*$ is also unitary
          $$|AU|_F^2 = |(U^*A^*)^*|_F^2 = |U^*A^*|_F^2 = |A^*|_F^2 = |A|_F^2$$





          Alternative argument:



          Note that $A^*A ge 0$ so there exists an orthonormal basis ${u_1, ldots, u_n}$ for $mathbb{C}^n$ such that $A^*A u_i = lambda_i u_i$ for some $lambda ge 0$.



          We have
          $$sum_{i=1}^n |Au_i|_2^2 = sum_{i=1}^n langle Au_i, Au_irangle = sum_{i=1}^n langle A^*Au_i, u_irangle = sum_{i=1}^n lambda_i =operatorname{Tr}(A^*A) = |A|_F^2$$



          because the trace is the sum of eigenvalues.



          The interesting part is that the sum $sum_{i=1}^n |Au_i|_2^2$ is actually independent of the choice of the orthonormal basis ${u_1, ldots, u_n}$. Indeed, if ${v_1, ldots, v_n}$ is some other orthonormal basis for $mathbb{C}^n$, we have
          begin{align}
          sum_{i=1}^n |Au_i|_2^2 &= sum_{i=1}^n langle A^*Au_i, u_irangle\
          &= sum_{i=1}^n leftlangle sum_{j=1}^nlangle u_i,v_jrangle A^*A v_j , sum_{k=1}^nlangle u_i,v_krangle v_krightrangle\
          &= sum_{j=1}^n sum_{k=1}^n left(sum_{i=1}^nlangle u_i,v_jrangle langle v_k,u_irangleright)langle A^*A v_j,v_krangle\
          &= sum_{j=1}^n sum_{k=1}^n langle v_j,v_kranglelangle A^*A v_j,v_krangle\
          &= sum_{j=1}^n langle A^*A v_j,v_jrangle\
          &= sum_{j=1}^n |Av_j|_2^2
          end{align}



          Now, if $U$ is unitary, for any orthonormal basis ${u_1, ldots, u_n}$ we have that ${Uu_1, ldots, Uu_n}$ is also an orthonormal basis so:



          $$|AU|_F^2 = sum_{i=1}^n |A(Ue_i)|^2 = |A|_F^2$$







          share|cite|improve this answer














          share|cite|improve this answer



          share|cite|improve this answer








          edited Jan 4 at 12:28

























          answered Jan 4 at 12:20









          mechanodroidmechanodroid

          27.1k62446




          27.1k62446












          • $begingroup$
            That's true, but the OP doesn't want to refer to trace.
            $endgroup$
            – A.Γ.
            Jan 4 at 12:26












          • $begingroup$
            @A.Γ. True, I missed that. Your argument is great. I have added an alternative argument which may be helpful, but it still uses the trace.
            $endgroup$
            – mechanodroid
            Jan 4 at 12:30


















          • $begingroup$
            That's true, but the OP doesn't want to refer to trace.
            $endgroup$
            – A.Γ.
            Jan 4 at 12:26












          • $begingroup$
            @A.Γ. True, I missed that. Your argument is great. I have added an alternative argument which may be helpful, but it still uses the trace.
            $endgroup$
            – mechanodroid
            Jan 4 at 12:30
















          $begingroup$
          That's true, but the OP doesn't want to refer to trace.
          $endgroup$
          – A.Γ.
          Jan 4 at 12:26






          $begingroup$
          That's true, but the OP doesn't want to refer to trace.
          $endgroup$
          – A.Γ.
          Jan 4 at 12:26














          $begingroup$
          @A.Γ. True, I missed that. Your argument is great. I have added an alternative argument which may be helpful, but it still uses the trace.
          $endgroup$
          – mechanodroid
          Jan 4 at 12:30




          $begingroup$
          @A.Γ. True, I missed that. Your argument is great. I have added an alternative argument which may be helpful, but it still uses the trace.
          $endgroup$
          – mechanodroid
          Jan 4 at 12:30


















          draft saved

          draft discarded




















































          Thanks for contributing an answer to Mathematics Stack Exchange!


          • Please be sure to answer the question. Provide details and share your research!

          But avoid



          • Asking for help, clarification, or responding to other answers.

          • Making statements based on opinion; back them up with references or personal experience.


          Use MathJax to format equations. MathJax reference.


          To learn more, see our tips on writing great answers.




          draft saved


          draft discarded














          StackExchange.ready(
          function () {
          StackExchange.openid.initPostLogin('.new-post-login', 'https%3a%2f%2fmath.stackexchange.com%2fquestions%2f3061577%2ffroebenius-norm-is-unitarily-invariant%23new-answer', 'question_page');
          }
          );

          Post as a guest















          Required, but never shown





















































          Required, but never shown














          Required, but never shown












          Required, but never shown







          Required, but never shown

































          Required, but never shown














          Required, but never shown












          Required, but never shown







          Required, but never shown







          Popular posts from this blog

          Can a sorcerer learn a 5th-level spell early by creating spell slots using the Font of Magic feature?

          Does disintegrating a polymorphed enemy still kill it after the 2018 errata?

          A Topological Invariant for $pi_3(U(n))$